PT8.S1.Q20 - Everyone at last week's neighborhood association meeting

MattLSATMattLSAT Core Member
edited June 2022 in Logical Reasoning 18 karma

Is anyone able to explain not just why AC B is correct, but why AC A is incorrect?

My understanding from the question stem is that the correct answer choice will be one that establishes either one of the two plans (demolition or rehabilitation). I chose AC A because I reasoned that the principle established that rehabilitation should occur UNLESS the neighborhood association deems the buildings a threat (which they do). So if the neighborhood deems the houses in question a threat, then they should not be rehabilitated, and if there are only two options: demolition or rehabilitation, then demolition would need to take place.

My guess as to why AC A is incorrect is because AC A only rules out rehabilitation as a possibility and does not establish that demolition will occur. AC B used some tricky language and it established that rehabilitation should occur and that makes sense, but unfortunately I read AC A first and understood it as establishing reason for demolition so I was biased in evaluating answer choice B. This question stem was especially odd so I am not very worried about getting confused by it but I would still like to understand this question better.

Comments

  • u______uu______u Alum Member
    edited June 2022 233 karma

    The most important part in B is "if the first proves unsatisfactory."
    This conditional gives the statement the flexibility to choose either option.
    It basically says:
    If you have two options and one of them prevents the other if the first choice is unsatisfactory, then choose the option that doesn't prevent the other option.
    This makes it so that if demolition was unsatisfactory, it would've prevented rehabilitation. Since that's the case, then you should've chosen rehabilitation.
    However, demolition wasn't unsatisfactory so you're completely fine accepting demolition as your option.
    A is far simpler: "choose the option that provides the most amount of housing unless it's a safety threat."
    Well, everyone thought it was a threat as stated in the stimulus so you would never choose the option that provides the most amount of housing. Therefore, it doesn't provide the same flexibility as B does.

Sign In or Register to comment.